one of the acute angles of a right angle triangle is 35 degree find the measure of the acute angle.​

Answers

Answer 1

Answer:

55°

Step-by-step explanation:

One triangle = 180° Therefore 90°+35°+ Unknown acute angle = 180°

125°+ Unknown acute angle = 180°

Unknown acute angle = 180°-125°

Unknown acute angle =55°

Answer 2

Answer:

55°

Step-by-step explanation:

One triangle 180° 90+35 =125-180=55°


Related Questions

if a chocolate cake is divided into six equal slices and each slice has 280 calories

Answers

Answer:

Whats the question?

Step-by-step explanation:

Answer:

1,680

Step-by-step explanation

6 slices each 280 calories, 280 x 6 = 1,680

e/kaid_274404147475337871026587/assig!
Khan Academy
Percent word problems
Of the 50 U.S. states, 4 have names that start with the letter W.
What percentage of U.S. states have names that start with the letter W?
0
Stuck? Review related articles/videos or use a hint.

Answers

Answer:

Step-by-step explanation:

4/50 = 0.08

0.08(100) = 8%

How many pounds are in 1 1/2 pounds and 8 ounces? There are _____ pounds in 1 1/2 pounds and 8 ounces.

Answers

Answer:

Hope this helps

Step-by-step explanation:

Since 16 ounces equal 1 pound, we’ll add 16 + 16, which is 32 ounces. For one half of a pound, simply divide the amount of ounces in a pound (16) by half. 16 divided by 2 is 8, so in total your answer would be 40 ounces in 2 1/2 pounds.

35. In her rough work, Janu added 2389 and 740 as shown here.
Is it okay to write the number 740 this way in the addition?
(Hint: You do NOT need to calculate)
(A) Yes, because 0 has no value so 74 is the same as 740.
(B) No, because the answer will be wrong if 0 is not written.
(C) No, because the digits 7 and 4 have NOT been arranged according to their value.
(D) Yes, because 7 is added with the hundred's digit and 4 with the ten's digit.

Answers

Answer:

D

Step-by-step explanation:

The width of a triangle is six more than twice the height. The area of the triangle is 88in2. Find the height and width of the triangle.

Answers

Answer:

w = 22 in, h = 8 in.

Step-by-step explanation:

Reading the problem, I immediately see two equations. This hints me towards a system of equations problem.

Statement 1 is The width of a triangle is six more than twice the height.

The width (w) of a triangle is (=) six (6) more (+) than twice (2) the height (h).

Equation 1: w = 6 + 2h

Statement 2 is The area of the triangle is 88 in2.

The formula for the area of a triangle is [tex]\frac{wh}{2}[/tex]. So, we get our second equation.

Equation 2: wh/2 = 88

We see that we have the same number of equations as unknowns, two. This is important as it lets us know that we can find two unknowns in the two equations. If there are more unknowns than equations, then we cannot find the values of all the variables. Look for another equation in the problem. However, if the # equations greater than or equal to number of unknowns, then we are ready to start solving!  So, we can now move to solving.

Equation 1 isolates one of the unknowns (w). So, we can replace w in the second equation with the right side of eq 1. Then, we simplify for h.

[tex]\frac{wh}{2} = 88[/tex]

[tex]\frac{(6 + 2h)h}{2} = 88[/tex]

[tex]\frac{6h + 2h^2}{2} = 88[/tex]

[tex]6h + 2h^2 = 176[/tex]

[tex]2h^2 + 6h - 176 = 0[/tex]

[tex]h = -11, 8[/tex] (assuming you know how to solve quadratics)

Since we know that heights of triangles cannot be negative, we find that [tex]h = 8[/tex] in.

For the final step, we plug [tex]h[/tex] into our first equation to find [tex]w[/tex].

[tex]w = 6 + 2h[/tex]

[tex]w = 6 + 2(8)[/tex]

[tex]w = 6 + 16[/tex]

[tex]w = 22[/tex]

So, the answer is w = 22 in, h = 8 in.

To check if we were right, we can plug the values into both equations to see if they are equal. I will skip equation 1 since we used that equation "as-is" to find w.

[tex]\frac{wh}{2} = 88[/tex]

[tex]\frac{22 * 8}{2} = 88[/tex]

[tex]11 * 8 = 88[/tex]

[tex]88 = 88[/tex]

The solution checks out!

geomtry plz help 15 points

Answers

Answer:

it should be 60°. the single marks on each side means they are all equal

Answer:

m∠L=60º

Step-by-step explanation:

The triangle is an equilateral triangle meaning that all angles and sides are equal.

m∠L=60º

Hope this helps :)

geomtry plzzz help 15 points

Answers

Answer:

19

Step-by-step explanation:

they give you xz, and they want half, so just divide 38 by 2

Suppose a jar contains 17 red marbles and 32 blue marbles. If you reach in the jar and pull out 2 marbles at random, find the probability that both are red.

Answers

There are 49 total marbles, the chance of you getting a red is 17/49, and if you don’t replace the marble and pull out another one, the probability of that being red is 17/49*16/48=17/147. That’s approximately a .12% chance.
16/48=1/3

The probability that both are red marbles = [tex]\bold{\frac{17}{147} }[/tex]

What is probability?

"It is finding out the possibilities of the occurrence of an event."

Formula to find the probability of an event:

"P(A) = n(A) / n(S)

where, n(A) is the number of favorable outcomes of an event A

n(S) is the total number of outcomes for an experiment"

For given example,

A jar contains 17 red marbles and 32 blue marbles.

If we pull out 2 marbles at random, we need to find the probability that both are red.

The number of possible outcomes,

[tex]\Rightarrow n(S)=^{49}C_2\\\\ \Rightarrow n(S)=\frac{49!}{2!(49-2)!}\\\\ \Rightarrow n(S)=1176[/tex]

The number of possible outcomes of selecting both the red marbles.

[tex]\Rightarrow n(A)=^{17}C_2\\\\ \Rightarrow n(A)=\frac{17!}{2!(17-2)!}\\\\ \Rightarrow n(A)=136[/tex]

The probability that both the marbles are red,

[tex]\Rightarrow P(A)=\frac{n(A)}{n(S)}\\\\ \Rightarrow P(A)=\frac{136}{1176}\\\\ \Rightarrow P(A)=\frac{17}{147}[/tex]

Therefore, the probability that both are red marbles = [tex]\bold{\frac{17}{147} }[/tex]

Learn more about the probability here:

brainly.com/question/11234923

#SPJ2

What’s the slope of 10,6 and 4, 1.2

Answers

let's firstly change the 1.2 to a fraction

[tex]1.\underline{2}\implies \cfrac{12}{1\underline{0}}\implies \cfrac{6}{5} \\\\[-0.35em] ~\dotfill\\\\ (\stackrel{x_1}{10}~,~\stackrel{y_1}{6})\qquad (\stackrel{x_2}{4}~,~\stackrel{y_2}{\frac{6}{5}}) \\\\\\ \stackrel{slope}{m}\implies \cfrac{\stackrel{rise} {\stackrel{y_2}{\frac{6}{5}}-\stackrel{y1}{6}}}{\underset{run} {\underset{x_2}{4}-\underset{x_1}{10}}}\implies \cfrac{~~ \frac{6-30}{5}~~}{-6}\implies \cfrac{~~ \frac{-24}{5}~~}{-6}\implies \cfrac{~~ -\frac{24}{5}~~}{-\frac{6}{1}}[/tex]

[tex]-\cfrac{\stackrel{4}{~~\begin{matrix} 24 \\[-0.7em]\cline{1-1}\\[-5pt]\end{matrix}~~}}{5}\cdot -\cfrac{1}{\underset{1}{~~\begin{matrix} 6 \\[-0.7em]\cline{1-1}\\[-5pt]\end{matrix}~~}}\implies \boxed{\cfrac{4}{5}}[/tex]

2x - 5y = 6 and -2x + 7y = 14

*Using elimination*​

Answers

2x-5y+ (-2x) + 7y= 6+14
2y = 20
y= 10
2x-5(10)=6
2x= 56
x= 28
here is the ans.hope this helps!

What is the coefficient of x in expression of 5mx​

Answers

Coefficient of this expression is 5 only.

- BRAINLIEST answerer

Evaluate this function: h(t) = |t+2| + 3; Find h(6)

Answers

Answer:

Step-by-step explanation:

h(6) = |6 + 2| + 3 = |8| + 3 = 8 + 3 = 11

plz help on a timed test will pay 10 points and make brainlest

What other countries have set up early earthquake warning systems? Which of these countries are in the Ring of Fire? Why is it important for these Ring of Fire countries to have warning systems?

Answers

Answer:

Mexico, Japan, South Korea, Taiwan, and the United States, Chile, Japan, the US west coast;  Chile, Japan, the US west coast; So all that is left for the people who live around the Ring of Fire is to be aware of the danger, perhaps to live further inland, build safer, earthquake-resistant housing, and for nations everywhere to improve oceanic and land-based early-warning systems to help minimize the risk to life

Step-by-step explanation:

please help this is for my study guide thanks! ​

Answers

The volume is 29.32 <3

Last year at a certain high school, there were 100 boys on the honor roll and 60 girls on the honor roll. This year, the number of boys on the honor roll decreased by 11% and the number of girls on the honor roll decreased by 10%. By what percentage did the total number of students on the honor roll increase? Round your answer to the nearest tenth (if necessary).​

Answers

Answer:

-11 boys -6 girls

Step-by-step explanation:

Is 40 is 50% of 80 true or false
With step by step evidence

Answers

Answer:

please mark brainliest

it is true

Step-by-step explanation:

if you multiplie 80 by 0.50 you will get 40,


A high school track team's long jump record is 22 ft 6 3/4in. This year, Arthur's best long jump is 21 ft 10 1/2in. If long jumps are measured to the nearest quarter inch, how much farther must
Arthur jump to break the record?
To beat the school long jump record,

Arthur must jump an additional ___inches.

(Type a whole number, fraction, or mixed number.)

Answers

Arthur must jump an additional 8.25 inches so as to break the record.

1 foot =  12 inches

Long jump record = 22 ft 6 3/4in = 22 ft 6.75 in = (22ft * 12 in per ft) + 6.75 in

Long jump record = 270.75 in

Arthur's long jump record = 21 ft 10 1/2in = 21 ft 10.5 in = (21ft * 12 in per ft) + 10.5 in

Arthur's long jump record = 262.5 ft.

To break the record, the distance needed to be jump by Arthur = 270.75 in - 262.5 ft. = 8.25 in

Arthur must jump an additional 8.25 inches so as to break the record.

Find out more at: https://brainly.com/question/20796404

what is 4% of 32? is it a percent, base, or amount

Answers

Answer:

1.28 - amount

Step-by-step explanation:

You would multiply 32 times 0.04 and get 1.28 which I assume would be an amount.

The amount of 4% of 32 is 1.28.

We have to determine what is 4% of 32.

What is the percentage?

This free percentage calculator computes a number of values involving percentages, including the percentage difference between two given values.

Suppose it is an x percent.

Therefore x multiply 32 times 0.04 and get 1.28 an amount.

To learn more about the percentage of visits:

https://brainly.com/question/24304697

#SPJ2

What is a quartic function with only two real zeros at x = 7 and x = 13
A. y = (x - 10) ^ 5 - 81 OR y = x ^ 5 - 20x ^ 4 + 92x ^ 3 + 34x ^ 2 - 20x + 91

B. Y= (x - 10) ^ 4 - 81 OR y = x^4 - 20x +91

C. y = (x - 3) ^ 4 - 41 OR y = 2x ^ 4 - 21x ^ 3 + 52x ^ 2 - 17x + 609

D. y = (x+ 8)^4 + 71 OR y = x^4 - 3x^3 + 7x^2 - 5x + 13

Answers

Zeros be a and b

a=7b=13

Function

[tex]\\ \sf\longmapsto y=x^2-(a+b)x+ab[/tex]

[tex]\\ \sf\longmapsto y=x^2-(7+13)x+(7)(13)[/tex]

[tex]\\ \sf\longmapsto y=x^2-20x+91[/tex]

Option B

g A long-term study has revealed that a test for cancer in men is very effective. The study shows that 89% of the men for which the test is positive actually have cancer. If a man selected at random tests positive for cancer with this test, what is the probability that he does not have cancer

Answers

The answer is 89% because I had that Same question and it was 89%

see question above ^^​

Answers

I think the answer is (4) because if you rotate AEC to the left (counterclockwise) and follow by the scale factor of 2 so, CE = 4x2 = 8 and CA = 3x2 = 6. Hope this help :))

What is 75.65 rounded to the nearest whole number?​

Answers

It is 76 as the other person said, what I like to think to help me remember is five or more raise the score, 4 or less, let it rest

in a supermarket, credit card purchases have an additional charge of 8% of the purchase made. If I bought 2 suitcases that cost s / .165.60 each and paid with a credit card, how much do I have to pay?

Answers

9514 1404 393

Answer:

  357.70

Step-by-step explanation:

The final cost will be ...

  (number of suitcases) × (cost per suitcase) × (1 +8%)

  = 2(165.60)(1.08) = 357.70

When it was 70 degrees outside. 50 members showed up at a beach club. For each degree the temperature rose, another 10 members came to the beach club. Write the function, Kt), that represents the number of members at the beach club as a function of the temperature​

Answers

Answer:

what to do here I don't know sorry

...............

.

.

........

.

.

...............

how to find the volume of an oven with the inside dimensions 3 ft by 1.5 ft by 3 ft

Answers

9514 1404 393

Answer:

  13.5 ft³

Step-by-step explanation:

The volume of a cuboid is the product of its length, width, and height:

  V = (3 ft)(1.5 ft)(3 ft) = 13.5 ft³

The volume of the oven is 13.5 ft³.

__

You may see this as the formula ...

  V = LWH

Juan wants to buy a video game for $63. He saves $12 every Friday. Part A. Create an equation to represent Juan's total savings, y, in dollars, after x Fridays. y​

Answers

Answer:

Answer in explanation.

Step-by-step explanation:

An equation to represent Juan's total savings y, in dollars, after x Fridays will be;

⇒ y = 12x

What is an expression?

Mathematical expression is defined as the collection of the numbers variables and functions by using operations like addition, subtraction, multiplication, and division.

Given that;

Juan wants to buy a video game for $63.

And, He saves $12 every Friday.

Now,

Let total dollars after x Friday = y

And, He saves $12 every Friday.

So, We can formulate;

⇒ Total money (y) = 12x

⇒ y = 12x

Thus, An equation to represent Juan's total savings y, in dollars, after x Fridays will be;

⇒ y = 12x

Learn more about the mathematical expression visit:

brainly.com/question/1859113

#SPJ2

Solve the equation: 3.017 + k = 5.134

Answers

Answer:

k=2.117

Step-by-step explanation:

Answer:

3.017 + k = 5.134

So, k = 5.134 - 3.017

So, k = 2.117


[tex]2 {}^{3} \times 2 - {}^{3} [/tex]
I need answers

Answers

The answer is 13 I might be wrong but I think it’s 13
The answer is 13

Step by step

2^3 x 2 - 3=
2 x2 x2=8
8 x2=16
16-3=13

which number will reach if we move 6 steps to the right -3

Answers

Answer:

If you meant -3 + 6, then the correct answer is 3.

You should put some extra effort in composing questions with all the required information, like a picture maybe.

Answer:

your answer has to be 3

Step-by-step explanation:

if you were to move to the right in number line you have to add so -3+6 is equals to 3

La probabilidad de que julio salga con carla es 0.75 , y la probabilidad de que salga con marisol es de 0.50. Si la probabilidad de que salga con carla o marisol es 0.85; calcular la probabilidad de que salga con ambas a la vez

Answers

La probabilidad de que Julio salga con Marisol y Carla al mismo tiempo es 0.40

What is the addition rule of probability for two events?

For two events A and B, we have:

Probability that event A or B occurs = Probability that event A occurs + Probability that event B occurs - Probability that both the event A and B occur simultaneously.

This can be written symbolically as:

[tex]P(A \cup B) = P(A) + P(B) - P(A \cap B)[/tex]

Given that;

The probability that Julio goes out with Carla is 0.75, and

the probability that he goes out with Marisol is 0.50.

The probability that he goes out with Carla or Marisol is 0.85

To find:

The probability that he will date both at the same time.

Let we take:

A = event that Julio goes with Carla

B = event that Julio goes with Marisol, then, the given expression is symbolically represented as:

[tex]P(A) = 0.75\\[/tex][tex]P(B) = 0.50[/tex][tex]P(A \cup B) = 0.85[/tex]

To find: [tex]P(A \cap B)\\[/tex]

Using the addition rule, we get:

[tex]P(A \cup B) = P(A) + P(B) - P(A \cap B)\\0.85 = 0.75 + 0.50 - P(A \cap B)\\P(A \cap B) = 1.25 - 0.85 = 0.40[/tex]

Thus, the probability that Julio will date Marisol and Carla at the same time is 0.40

Learn more about probability here:

brainly.com/question/1210781

The probability that Julio will date Marisol and Carla at the same time is 0.40.

Here, we have, For two events A and B, we have:

Probability that event A or B occurs = Probability that event A occurs + Probability that event B occurs - Probability that both the event A and B occur simultaneously.

This can be written symbolically as:

P(A ∪ B) = P(A) + P(B) - P(A ∩ B)

Given that;

The probability that Julio goes out with Carla is 0.75, and

the probability that he goes out with Marisol is 0.50.

The probability that he goes out with Carla or Marisol is 0.85

To find:

The probability that he will date both at the same time.

Let we take:

A = event that Julio goes with Carla

B = event that Julio goes with Marisol, then, the given expression is symbolically represented as:

P(A) = 0.75

P(B) = 0.50

P(A∪B)= 0.85

To find: P(A ∩ B)

Using the addition rule, we get:

P(A ∪ B) = P(A) + P(B) - P(A ∩ B)

0.85 = 0.75 + 0.50 - P(A ∩ B)

P(A ∩ B) = 0.40

Thus, the probability that Julio will date Marisol and Carla at the same time is 0.40.

Learn more about probability here:

brainly.com/question/1210781

#SPJ3

complete question:

The probability that Julio dates Carla is 0.75, and the probability that he dates Marisol is 0.50. If the probability that he goes out with carla or marisol is 0.85; Calculate the probability that he will date both at the same time.

Other Questions
HELP WILL GIVE BRAINLIEST Can someone help me? What would happen to the dome ecosystem without bacteria an fungi The _______ is responsible for fetching program instructions, decoding each one, and performing the indicated sequence of operations. The formation of children's sense of self is most strongly influenced by: Group of answer choices The self-concepts of their parents Their inherited temperaments How other people treat them Their athletic ability 15 points for anwserDetermine whether each expression is equivalent to x^(7/4). Expression Yes No(7&x^4 ) (x^7 ) (x)^7 (x^(7/4) ) (x^5 )(x^2 ) (5&x^4 )(2&x^4 ) (x)^7/(x)^0 Help. If you were a hackathon team manager, how could you best address the conflict created by having more volunteers than open roles A 4-mile cab ride cost $5.50, and a 9 mile cab ride costs $8.00. Identify two points from this information. Let the total cost of the cab ride represent (y) and the number of miles traveled (x) Dave owns 15 shares of ABC Mining stock. On Monday, the value ofeach share rose $2, but on Tuesday the value fell $5. What is thechange in the value of Dave's shares? To travol thoWrite an expression to represent. Find the domain and range Why did the Italians and Germans support Francisco Franco in the Spanish Civil War? A. They wanted to spread communism throughout the rest of Europe. B. They wanted to develop new equipment and strategies to use in future wars C. They admired Franco for his democratic values and wanted him to come into office O D. They despised monarchies and wanted the Spanish king to fall. su HELP ME WITH THIS PELASE! GET IT RIGHT GET IT BRAINLIEST!!!! If you see such unnecessary question like this, please report it. This person always asks others for bad things. Please report him. That's your responsibility as a polite Brainly User. Thank You ! did ashoka do more as a military leader or a buddhist emporor A paragraph summary about industrialization BIG IF....If you had the power to change something in the world right now, big or small, what would it be? Why? Mikey's family is taking a road trip. Their car can travel no more than 260 miles without needing to refill the gas tank. They have traveled 112 miles since they last filled the gas tank.Mikey knows that the inequality representing this situation is 112 + x 260.Select from the drop-down menu to correctly interpret the solution of this inequality.Mikey's family can drive_____more miles before they need to refill the gas tank.A. at most 148B. at least 148C. more than 372D. more than 372 PARAGRAPH 1: What is energy? Why is energy important? Describe at least two forms of energy. PARAGRAPH 2: What is thermal energy? Describe how thermal energy can move between objects and in what direction (hot to cold or cold to hot) . Include at least one example of how thermal energy moves. Do NOT make a list. Write it as a paragraph!A complete response to the writing prompt is 2 paragraphs. Each paragraph must include at least 3 sentences in order for you to receive credit. If you help me I'll mark you as a brain list WILL GIVE 100PTSFind the slope of a line that passes through (-2, -3) and (1, 1). A. 1 /3 B. 1 C. 2 D. 4/3